LSAT and Law School Admissions Forum

Get expert LSAT preparation and law school admissions advice from PowerScore Test Preparation.

 Administrator
PowerScore Staff
  • PowerScore Staff
  • Posts: 8916
  • Joined: Feb 02, 2011
|
#36962
Complete Question Explanation

Flaw in the Reasoning. The correct answer choice is (D)

The advertisement presented here inappropriately presumes that, because each winner of the Economic
Merit Prize for the past 25 years has been covered by the Acme retirement plan, these winners believe
that the plan offers them an economically secure future. The author concludes that the Acme plan is
good for those with similar needs.

In moving from the initial premise about coverage to the intermediate conclusion about security, the
advertisement makes the unwarranted assumptions that the winners of the Economic Merit Prize had
some control over the choice of their retirement plans, and that this choice was based in large part on
their needs (and not, for example, on sponsorship).

Answer choice (A): The stimulus provides information on the retirement plans of the winners over the
past 25 years. Information about plans from longer than 25 years ago is most likely irrelevant, and to
ignore it is not a flaw.

Answer choice (B): The advertisement argues that the Acme retirement plan is a good plan, not that it
is the only good plan, so this choice seems to misinterpret the argument as presented and is incorrect.
When a stimulus concludes that something is a solution, it is not vulnerable to the criticism that there
exist other solutions.

Answer choice (C): Since the winners of the Economic Merit Prize are never said to believe that the
Acme plan is good for all those similar to themselves, those economists are not portrayed as endorsing
the argument’s main conclusion.

Answer choice (D): This is the correct answer choice. If the winners of the Economic Merit Prize did
not select the Acme retirement plan themselves, the intermediate conclusion that they believe the plan is
good does not follow from the argument, and the argument falls apart. Who selected the plan was never
addressed, and this represents a flaw.

Answer choice (E): The argument in the stimulus makes no such assumption, so this choice is incorrect.
The advertisement was geared to those with “similar” needs, not those with “identical” needs.
 voodoochild
  • Posts: 185
  • Joined: Apr 25, 2012
|
#6044
Experts,
Why is E) incorrect?

The underlying reasoning behind recommending the plan is that the financial needs are identical. Isn't it? Why would I recommend a plan targeted for a 50-year old to a 21-year old guy? Their financial needs are different. 50 year old will invest more in bonds. 21 year old can go with equities. Not sure about E). I also see a gap in D). Still not clear.

To me, D sounded wrong because the argument says "economists have THUS CLEARLY recognized that plans offer them a financially secure future" -- clearly recognized => they did the analysis and chose the plan. Isn't it? It could be argued that what "thus" is. Not sure.

Please help. I am a bit lost.

Thanks
 Nikki Siclunov
PowerScore Staff
  • PowerScore Staff
  • Posts: 1362
  • Joined: Aug 02, 2011
|
#6046
The argument is structured as follows:

Premise: Each of the EMP winners from the past 25 yrs is covered by Acme.

Sub. Conclusion: The EMP winners have recognized that the Acme plan offers them a financially secure future.

Conclusion: It is probably a good plan for anyone with retirement needs similar to theirs.

First off, you should not take the sub. conclusion as a factual premise. Although it supports the main conclusion, it is a claim which is itself supported by the observation described in the first sentence (notice the conclusion indicator "thus clearly"), which makes it potentially suspect. Indeed, whether the EMP winners have actually recognized Acme as an awesome plan is far from clear, as is the conclusion that the Acme plan would work for anyone with retirement needs similar to theirs.

(D) describes an unwarranted assumption and is therefore the correct answer to this Flaw question. If the EMP winners didn't deliberately choose Acme, then we can no longer claim that they have "clearly recognized" Acme to be a great plan. As a domino effect, the main conclusion would be weakened as well, showing that (D) describes an assumption upon which the argument depends.

As far as (E) is concerned, this is not an assumption for the argument. The author clearly limits its target audience to "anyone with retirement needs similar to [the EMP winners']," and so cannot be faulted for assuming that everyone's retirement needs are identical to those of the EMP winners. This allegation would only be justified if the conclusion were a broad claim such as, "the Acme plan is a good plan for anyone." Also, note that the conclusion specifies "similar needs," not "identical needs."

Hope this clear things up! Let me know.
 emilysnoddon
  • Posts: 64
  • Joined: Apr 22, 2016
|
#25166
Hello,

I had trouble with this question. I was struggling between answer choices D and C and eventually chose C. My problem with answer choice D was that I felt as though even if all the winners of the Economic Merit Prize had the Acme retirement plan chosen for them by their employers that would not diminish the fact that they felt the plan offers them a financially secure future. I did not see why having some winners self select the plan was important. I feel as though I can still think that my 401K plan that my office set up for me my future, even though I didnt select the plan myself.

Could you please help me see where I went wrong with this question?

Thank you,

Emily
 David Boyle
PowerScore Staff
  • PowerScore Staff
  • Posts: 836
  • Joined: Jun 07, 2013
|
#25383
emilysnoddon wrote:Hello,

I had trouble with this question. I was struggling between answer choices D and C and eventually chose C. My problem with answer choice D was that I felt as though even if all the winners of the Economic Merit Prize had the Acme retirement plan chosen for them by their employers that would not diminish the fact that they felt the plan offers them a financially secure future. I did not see why having some winners self select the plan was important. I feel as though I can still think that my 401K plan that my office set up for me my future, even though I didnt select the plan myself.

Could you please help me see where I went wrong with this question?

Thank you,

Emily

Hello Emily,

"Each of the Economic Merit Prize winners . . . is covered by the Acme retirement plan. Since the winners . . . have thus clearly recognized that the Acme plan offers them a financially secure future . . ." asserts a link between the coverage and the winners' recognition of the plan's value. It seems sort of a stretch to say that they just coincidentally happen to recognize its value, even though someone else chose the plan for them. It makes more sense to assume, as in answer choice D, that the stimulus assumes at least some of them actively chose the plan.

Hope this helps,
David
 maximbasu
  • Posts: 59
  • Joined: May 19, 2016
|
#26627
Hi,

Would you please clarify this question a bit more.

I chose A instead of D. My issue with D is that the stimulus states that the winners clearly recognized the plan while D mirrors that; it states that some winners deliberately selected the plan (instead of employers doing that for them). Should I assume that some then didn't choose the plan? The focus of D is on deliberately selecting the plan, and that's throwing me off I assume.

MB
 Clay Cooper
PowerScore Staff
  • PowerScore Staff
  • Posts: 241
  • Joined: Jul 03, 2015
|
#26630
Hi maximbasu,

Thanks for your question.

The focus of the correct answer choice is indeed on whether the EMP winners deliberately chose the plan for themselves. We don't actually know that they did, and that is the biggest flaw in the argument.

The key here is to see that the subordinate (a.k.a. intermediate) conclusion is flawed and thus causes the overall conclusion to be flawed. The subordinate conclusion claims that it is clear that the EMP winners recognize the value of Acme plan, but we can't actually prove that - all we know is that they are covered by it. It could be (as the answer choice suggests) that the plan was chosen for them by their employers. It could even be the case that the Acme company is clever and deceitful and has given these people coverage under the plan free of charge, just to be able to claim them as plan members.

Either of these examples illustrates the flaw here: the fact that the last 25 winners all are covered by the same retirement plan does not necessarily prove that they all think it is valuable.

I hope that helps!
 jlkramer
  • Posts: 1
  • Joined: Jan 10, 2017
|
#31967
Hi,

Thank you for the explanations for why D is correct. However, I'm having a hard time figuring out why C is not also correct. The argument seems to proceed by claiming that since award winning economists use the plan, so should others with similar financial needs. Which to me would imply that in order for that statement to have any persuasive value, it would have to be assumed that the economists would be experts in attaining financially secure futures. While this may be true, it is not necessarily true. The past 25 EMP winners could have specialized in areas of economics that would not lead to expertise in attaining financially secure futures. If this were the case, then relying on their supposedly expert opinions would make the argument vulnerable to criticism.

Thanks for your help!

Jake
 David Boyle
PowerScore Staff
  • PowerScore Staff
  • Posts: 836
  • Joined: Jun 07, 2013
|
#31985
jlkramer wrote:Hi,

Thank you for the explanations for why D is correct. However, I'm having a hard time figuring out why C is not also correct. The argument seems to proceed by claiming that since award winning economists use the plan, so should others with similar financial needs. Which to me would imply that in order for that statement to have any persuasive value, it would have to be assumed that the economists would be experts in attaining financially secure futures. While this may be true, it is not necessarily true. The past 25 EMP winners could have specialized in areas of economics that would not lead to expertise in attaining financially secure futures. If this were the case, then relying on their supposedly expert opinions would make the argument vulnerable to criticism.

Thanks for your help!

Jake

Hello Jake,

C is not too bad an answer, true, though D is a better answer. Among other things, is it likely that all 25 winners didn't study fields of economics with expertise in attaining financially secure futures? That might be a stretch. Maybe all 25 are experts in financial security. So, C has some appeal, but may not be extremely strong.
And of course, we always look for the best answer, even if another answer has some merit to it. Thus, D is the one.

Hope this helps,
David
 kuma-turtle
  • Posts: 14
  • Joined: Aug 07, 2017
|
#38167
David Boyle wrote:
emilysnoddon wrote:Hello,

I had trouble with this question. I was struggling between answer choices D and C and eventually chose C. My problem with answer choice D was that I felt as though even if all the winners of the Economic Merit Prize had the Acme retirement plan chosen for them by their employers that would not diminish the fact that they felt the plan offers them a financially secure future. I did not see why having some winners self select the plan was important. I feel as though I can still think that my 401K plan that my office set up for me my future, even though I didnt select the plan myself.

Could you please help me see where I went wrong with this question?

Thank you,

Emily

Hello Emily,

"Each of the Economic Merit Prize winners . . . is covered by the Acme retirement plan. Since the winners . . . have thus clearly recognized that the Acme plan offers them a financially secure future . . ." asserts a link between the coverage and the winners' recognition of the plan's value. It seems sort of a stretch to say that they just coincidentally happen to recognize its value, even though someone else chose the plan for them. It makes more sense to assume, as in answer choice D, that the stimulus assumes at least some of them actively chose the plan.

Hope this helps,
David
Hello,

I feel terrible to still not completely understand why D is correct after all these discussions, but I just cannot see how D's assumption is important. Yes, I understand that just because the 25 are covered by the plan doesn't mean they recognize its value. Yet, how come choosing a plan for yourself and having it chosen for you makes a difference in recognizing its value? Wouldn't recognizing the value of a plan depend on the content of the plan, and not on who chose it?

Get the most out of your LSAT Prep Plus subscription.

Analyze and track your performance with our Testing and Analytics Package.